Applying the chain rule using trees?

Click For Summary
The discussion revolves around applying the chain rule using dependency trees to find the expression for ∂z/∂r. The initial attempt resulted in only two terms being calculated instead of the expected eight, indicating a misunderstanding of the tree structure. Participants noted that a correct approach should yield four terms, with one term involving the partial derivative of z with respect to y. The discussion emphasizes the importance of correctly identifying all dependencies and paths in the tree to achieve the necessary number of leaves. Properly following the chain rule is crucial for accurately determining the derivatives in complex functions.
Pizzerer
Messages
10
Reaction score
0

Homework Statement


Write out a tree (this will be a big tree) of dependencies and hence write down an expression for
∂z/∂r

Homework Equations


z=k(x, y)=xy2, x=(w1)(w2)+w3, y=w4;

w1=t, w2=t2, w3=2t+1, w4=sin(t);
t=r2+2s2

The Attempt at a Solution


This is the tree I drew and followed the relevant paths in order to try and write and expression but my expression is to short. I only calculate 2 terms when there is suppose to be 8:
http://www.picpaste.com/Untitled_Image-OZCiTMvb.jpg

This is wrong however. I need 16 leaves at the base. What am I doing wrong?
 
Last edited by a moderator:
Physics news on Phys.org
Pizzerer said:

Homework Statement


Write out a tree (this will be a big tree) of dependencies and hence write down an expression for
∂z/∂r

Homework Equations


z=k(x, y)=xy2, x=(w1)(w2)+w3, y=w4;

w1=t, w2=t2, w3=2t+1, w4=sin(t);
t=r2+2s2

The Attempt at a Solution


This is the tree I drew and followed the relevant paths in order to try and write and expression but my expression is to short. I only calculate 2 terms when there is suppose to be 8:
http://www.picpaste.com/Untitled_Image-OZCiTMvb.jpg

This is wrong however. I need 16 leaves at the base. What am I doing wrong?

I drew a diagram and got the same one you did.

As far as your partial ## \partial z/\partial r## is concerned, I get four terms to your two. Three of the four terms look like this:
$$ \frac{\partial z}{\partial x} \frac{\partial x}{\partial w_i} \frac{d w_i}{dt} \frac{\partial t}{\partial r}$$

For the fourth one, the first factor is the partial of z with respect to y.
 
Last edited by a moderator:
attachment.php?attachmentid=58345&stc=1&d=1367266069.jpg
 

Attachments

  • Untitled_Image-OZCiTMvb.1367266012.jpg
    Untitled_Image-OZCiTMvb.1367266012.jpg
    15.7 KB · Views: 481
Question: A clock's minute hand has length 4 and its hour hand has length 3. What is the distance between the tips at the moment when it is increasing most rapidly?(Putnam Exam Question) Answer: Making assumption that both the hands moves at constant angular velocities, the answer is ## \sqrt{7} .## But don't you think this assumption is somewhat doubtful and wrong?

Similar threads

  • · Replies 3 ·
Replies
3
Views
2K
  • · Replies 9 ·
Replies
9
Views
2K
  • · Replies 4 ·
Replies
4
Views
2K
  • · Replies 5 ·
Replies
5
Views
2K
  • · Replies 12 ·
Replies
12
Views
3K
  • · Replies 3 ·
Replies
3
Views
2K
Replies
2
Views
2K
  • · Replies 7 ·
Replies
7
Views
2K
  • · Replies 5 ·
Replies
5
Views
2K
  • · Replies 4 ·
Replies
4
Views
2K